The only way to learn MATHEMATICS

is to do MATHEMATICS.

- Paul Halmos -
СОНИРХОЛТОЙ БОДЛОГО
Математик сонирхогч хүүхэд, залуустаа зориулаад сонирхолтой, сургамжтай бодлогуудыг бие даан бодох байдлаар оруулж байх болно. Өөртөө зав гаргаад оролдож үзээрэй. Бодолтыг 7 хоногийн дараа оруулах бөгөөд түүнээс өмнө өөрсдийн бодолтуудаа seluunsanaa@gmail.com хаягаар ирүүлээрэй. Оновчтой, гоё бөгөөд нийтлэгчийн санаатай давхцаагүй бодолт ирвэл таны бодолтыг таны нэртэйгээр нийтлэх болно.
Бодолт илгээхдээ гаргацтай бичсэн бодолтоо тод, чанартайгаар зургийг авч явуулна. Өөрийн нэр болон сургууль, анги, утасны дугаараа бичээрэй.
БОДЛОГО №14
Дурын натурал $n$ тооны хувьд $$\sqrt[n]n< 1+\sqrt{\dfrac2n}$$ тэнцэтгэл биш биелэхийг батал.

Бодолт 1.
8x100 (iMath.mn)-н бодолт
Хоёр талыг $n$ зэрэг дэвшүүлээд $$n< \left(1+\sqrt{\dfrac2n}\right)^n$$ гэж батлахад болно. Баруун талыг биномын томьёогоор задлавал $$1+n\sqrt{\dfrac2n}+\dfrac{n(n-1)}2\cdot\dfrac2n+...=$$ $$=1+\sqrt{2n}+(n-1)+...>n$$ болж батлагдана.
ӨМНӨХ БОДЛОГУУД

Бодлогын бүтэн өгүүлбэр. Харгалзан $O_1,\ O_2$ төвүүд бүхий үл огтлолцох $\omega_1,\ \omega_2$ тойргууд өгөгдөв. $O_1$ цэгээс $\omega_2$ тойрогт татсан $O_1A$, $O_1B$ шүргэгчүүд $\omega_1$ тойрогтой $MN$ цэгүүдээр, $O_2$ цэгээс $\omega_1$ тойрогт татсан $O_2P$, $O_2Q$ шүргэгчүүд $\omega_2$ тойрогтой $RS$ цэгүүдээр огтлолцоно. Тэгвэл $MN=RS$ гэж батал.
Бодолт 1.
8x100 (iMath.mn)-н бодолт
Зурагт байгаагаар тэмдэглэгээнүүдийг хийе. $O_1O_2$ нь $\angle PO_2Q$ ба $AO_1B$ өнцгүүдийн биссиктрис гэдгээс $MN\perp O_1O_2\perp RS$ байна. Иймд, $\triangle O_1MX\sim\triangle O_1O_2A$ ба $\triangle O_2RY\sim \triangle O_2O_2P$ болно. Төсөөгийн харьцаа бичвэл $$\dfrac{MX}{r_2}=\dfrac{r_1}{O_1O_2},\ \dfrac{RY}{r_1}=\dfrac{r_2}{O_1O_2}\Rightarrow $$ $$\Rightarrow MX=\dfrac{r_1r_2}{O_1O_2}=RY$$ болох бөгөөд $MN=2MX$, $RS=2RY$ тул батлагдана.

Бодлогын бүтэн өгүүлбэр. $a$, $b$, $c$ нь гурвалжны талууд бөгөөд $a^2+b^2=kc^2$ бол $k>\dfrac12$ гэж батал.
Бодолт 1.
8x100 (iMath.mn)-н бодолт
Эсрэгээс нь $0\le k\le \dfrac12$ байх гурвалжин олддог гэе. Түүний $c$ талын эсрэг орших өнцгийг $\gamma< 180^\circ$ гээд косинусын теорем бичвэл $$a^2+b^2=kc^2=k(a^2+b^2-2ab\cos\gamma)\Rightarrow$$ $$\Rightarrow2kab\cos\gamma=(k-1)(a^2+b^2)\Rightarrow$$ $$\Rightarrow \cos\gamma=-\dfrac{(1-k)(a^2+b^2)}{k\cdot2ab}\le-\dfrac{(1-k)}k$$ гэж гарна. Энд $a,b\ge0$ бол $a^2+b^2\ge2ab$ буюу $\dfrac{a^2+b^2}{2ab}\ge1$ гэдгийг ашиглав. Одоо $$\dfrac{1-k}k\ge1\Leftrightarrow 0\le k\le\dfrac12$$ байх тул $\cos\gamma\le-1$ болж зөрчилд хүрнэ.
Бодолт 2.
И.Энхдаржаа (Олонлог төв, 8б анги)-н бодолт
Кошийн тэнцэтгэл бишээр $$a^2+b^2\ge2ab\Rightarrow$$ $$\Rightarrow 2a^2+2b^2\ge (a+b)^2$$ гэж гарна. Гурвалжны тэнцэтгэл бишээр $a+b>c$ гэдгээс $$(a+b)^2>c^2$$ байна. Эдгээрээс $$2a^2+2b^2>c^2\Rightarrow$$ $$\Rightarrow a^2+b^2>\dfrac12 c^2$$ байх бөгөөд $$a^2+b^2=kc^2$$ байхын тулд $k>\dfrac12$ байх нь батлагдав.

Бодлогын бүтэн өгүүлбэр. Өндөр нь $2$, суурь нь $6$ нэгж бүхий ямар ч гурвалжныг $3$-аас олонгүй шулуунаар огтлоод, үүссэн хэсгүүдээр нэгж кубыг бүрэн хучиж чадна гэж батал.
Бодолт 1.
8x100 (iMath.mn)-н бодолт
Өгөгдсөн нөхцөлийг хангах ямар ч гурвалжныг 2-оос олонгүй шулуунаар огтлоод $1\times6$ хэмжээстэй тэгш өнцөгт үүсгэж чадахыг дараах зургуудаас хараарай.

Үүссэн $1\times6$ тэгш өнцөгтийг голоор нь 1 удаа хувааж

дурсийг бүрэн хужич чадна. Энэ нь нэгж кубын дэлгээс болох тул батлагдав.

Бодлогын бүтэн өгүүлбэр. $k$ ба $l$ нь натурал тоонууд бол $36^k-5^l$ илэрхийллийн авч болох хамгийн бага эерэг утгыг ол.
Бодолт 1.
8x100 (iMath.mn)-н бодолт
$36^k$ үргэлж $6$-аар, $5^l$ нь үргэлж $5$-аар төгсөнө. Иймд, $36^k-5^l$ тоо үргэлж $1$-ээр төгсөнө. $k=1,\ l=2$ үер илэрхийллийн утга $11$ байна. Одоо $11$-ээс бага утга авах боломжгүйг баталъя. $1$-ээр төгсөх $11$-ээс бага тоо бол зөвхөн $1$ юм. Иймд $$36^k-5^l\neq1$$ гэж батална. Эсрэгээс нь $36^{k_0}-5^{l_0}=1$ байх $k_0,\ l_0\in\mathbb N$ тоонууд олддог гэе. Тэгвэл $36^{k_0}\equiv0\pmod4$ ба $5^{l_0}\equiv 1\pmod4$ тул $$36^{k_0}-5^{l_0}\equiv-1\equiv1\pmod4$$ болж зөрчилд хүрнэ. Өөрөөр хэлбэл, $36^k-5^l$ илэрхийлэл хэзээ ч $1$-тэй тэнцэж чадахгүй. Иймд, хамгийн бага эерэг утга нь $11$ байна.

Бодлогын бүтэн өгүүлбэр. Аравтын бичлэг нь зөвхөн сондгой цифр агуулсан бүтэн квадрат болдог хамгийн их тоог ол.
Бодолт 1.
8x100 (iMath.mn)-н бодолт
Бодлогын нөхцөлийг хангах 2-оос багагүй оронтой тоо олохыг зорьё. Бидний олох тоо $$A=10x+y$$ тооны квадрат болдог гэе. Бодлогын нөхцөлөөс $y=1,3,5,7,9$ байх ёстой. $$\begin{array}{c|ccccccccc } y&1&3&5&7&9\\ \hline y^2&1&9&25&49&81\\ \end{array}$$ байхыг анзаарвал $$A^2=100x^2+20xy+y^2$$ тооны аравтын орны цифр тэгш байх нь гарцаагүй юм. Иймд, бодлогын нөхцөлийг хангах тоонууд нь ердөө $1,\ 9$ байна.

Бодлогын бүтэн өгүүлбэр. $1^3+2^3+...+n^3$ тооны нэгжийн орны цифр 1 бол $1+2+...+n$ тооны нэгжийн орны цифр хэд байх вэ?
Бодолт 1.
8x100 (iMath.mn)-н бодолт
Дурын натурал $n$ тооны хувьд $$1^3+2^3+...+n^3=(1+2+...+n)^2$$ байдаг ба $1+2+...+n=\dfrac{n(n+1)}2$ тул $\left[\dfrac{n(n+1)}2\right]^2$ тоо 1 нэгээр төгсөнө гэсэн үг. Эндээс, $\dfrac{n(n+1)}2$ тоо 1-ээр эсвэл 9-р төгсөх ёстой гэж гарна. Гэвч $\dfrac{n(n+1)}2$ тоо 9-өөр төгсөх боломжгүй гэдгийг баталъя. Эсрэгээс нь боломжтой гэж үзвэл $n(n+1)$ тоо 8-аар төгсөх ёстой болно. Дараалсан тоонуудын үржвэр 8-аар төгсөх боломжгүй гэдгийг доорх үржвэрүүдээс харж болно. $$0\cdot1=0,\ 1\cdot2=2,\ 2\cdot3=6$$ $$3\cdot4=12,\ 4\cdot5=20,\ 5\cdot6=30$$ $$6\cdot7=42,\ 7\cdot8=20,\ 8\cdot9=30$$ Иймд, $\dfrac{n(n+1)}2$ тоо 1-ээр төгсөх ганц л боломжтой.

Бодлогын бүтэн өгүүлбэр. Адил хажуут $ABC$ тэгш өнцөгт гурвалжны $CA$, $CB$ катетууд дээр $D$ ба $E$ цэгүүдийг $CD=CE$ байхаар авав. $C$ ба $D$ цэгүүдээс $AE$ шулуунд татсан перпендикуляр шулуун $AB$ талтай харгалзан $L$, $K$ цэгүүдээр огтлолцоно. $KL=LB$ байхыг батал.

Бодолт 1.
8x100 (iMath.mn)-н бодолт
$B$ оройг дайруулан $CL$-тэй параллель шулуун татаад $AC$ талын үргэлжлэлтэй огтлолцох цэгийг $F$ гэж тэмдэглэе. Тэгвэл $\triangle BFC\sim \triangle AEC$ байх бөгөөд төсөөгийн харьцаа бичвэл $$\dfrac{FC}{CE}=\dfrac{CB}{CA}=1\Rightarrow$$ $$\Rightarrow FC=CE=CD$$ болно.

$\angle FAB$-н хувьд фалесийн теоремоор $$\begin{cases}FC=CD\\ FB\parallel CL\parallel DK\end{cases}\Rightarrow BL=LK$$ болж батлагдана.

Бодлогын бүтэн өгүүлбэр. $P(x)+1$ нь $(x-1)^3$-д хуваагддаг, $P(x)-1$ нь $(x+1)^3$-д хуваагдах байх таван зэргийн $P(x)$ олон гишүүнтүүдийг ол.
Бодолт 1.
8x100 (www.iMath.mn)-н бодолт
$P(1)=-1$, $P(-1)=1$ байх нь ойлгомжтой. Өгсөн нөхцөлөөс $$P(x)+1=(x-1)^3Q_1(x)$$ $$P(x)-1=(x+1)^3Q_2(x)$$ байх $Q_1(x),\ Q_2(x)$ олон гишүүнтүүд олдоно. Энэ хоёр тэнцэтгэлээс уламжлал авбал $$P'(x)=(x-1)^2Q_1^*(x)$$ $$P'(x)=(x+1)^2Q_2^*(x)$$ байна гэдгээс $$P'(x)=A(x^2-1)^2=A(x^4-2x^2+1)$$ байх $A$ тоо олдоно. Эндээс $$P(x)=A\left(\dfrac{x^5}5-\dfrac{2x^3}3+x\right)+c$$ бөгөөд $P(1)=-1$, $P(-1)=1$ нөхцөлүүдээ орлуулвал $A=-\dfrac{15}8$, $c=0$ гэж олдоно.
Бодолт 2.
О. Лхагва (Увс, Тариалан, 1-р сургууль 11а анги)-н бодолт
$P(x)=ax^5+bx^4+cx^3+dx^2+ex+f$ гэе. $$(x-1)^3\ \big|\ P(x)+1$$ байхын тулд $$x-1\ \big|\ P(x)+1$$ $$x-1\ \big|\ (P(x)+1)'=P'(x)$$ $$x-1\ \big|\ (P(x)+1)''=P''(x)$$ байх ёстой. Үүнтэй адилаар $$x+1\ \big|\ P(x)-1$$ $$x+1\ \big|\ (P(x)-1)'=P'(x)$$ $$x+1\ \big|\ (P(x)-1)''=P''(x)$$ байна. Эдгээр дээр Безугийн теорем ашиглавал $$\begin{cases} a+b+c+d+e+f+1=0\\ -a+b-c+d-e+f-1=0\\ 5a+4b+3c+2d+e=0\\ 20a+12b+6c+2d=0\\ -20a+12b-6c+2d=0 \end{cases}$$ систем үүсэх бөгөөд энэ системийн шийд нь $a=-\dfrac38$, $b=d=f=0$, $c=\dfrac54$, $e=-\dfrac{15}8$ байна.
Бодолт 3.
Т. Билгүүн (УБ, СБД, 11-р сургууль 12-р анги)-н бодолт
Эхний нөхцөлөөс $$P(x)=(x-1)^3(ax^2+bx+c)-1$$ байг. Харин хоёрдугаар нөхцөлөөс $$P(x)=(x+1)^3(ax^2+mx+n)+1$$ байх $m,\ n$ тоонууд олдоно. Энэ хоёр адилтгалыг задалж, тэнцүүлэх бөгөөд коэффициентүүдийг тааруулан бодвол $$\begin{cases}b-3a=m+3a\\ c-3b+3a=n+3m+3a\\ -3c+3b-a=3n+3m+a\\ 3c-b=3n+m\\ -c-1=n+1 \end{cases}$$ систем үүснэ. Үүнийг бодоод $a=-\dfrac38$, $c=\dfrac54$, $e=-\dfrac{15}8$ гэж олно.
Бодолт 4.
Ч. Батзаяа (Дорноговь, Сайншанд, 1-р сургуулийн багш)-н бодолт
$P(x)=ax^5+bx^4+cx^3+dx^2+ex+f$ гэе. Шууд $P(x)+1$-г $x-1$-д хуваавал \begin{align*}&ax^4+(a+b)x^3+(a+b+c)x^2+\\ &+(a+b+c+d)x+\\ &+(a+b+c+d+e)\qquad(1)\end{align*} ноогдоод $a+b+c+d+e+f+1$ үлдэнэ. Хуваагдах ёстой тул $$a+b+c+d+e+f+1=0$$ байна. Одоо $(1)$-г $x-1$-д хуваавал \begin{align*}&ax^3+(2a+b)x^2+\\ &+(3a+2b+c)x+\\ &+4a+3b+2c+d\qquad (2)\end{align*} ноогдоод $5a+4b+3c+2d+e$ үлдэнэ. Эндээс $$5a+4b+3c+2d+e=0$$ тэнцэтгэл гаргаж авна. Үүнтэй адилаар $(2)$-г $x-1$-д хуваах замаар $$10a+6b+3c+d=0$$ тэнцэтгэлийг гаргаж авна. Энэ зарчмийн дагуу $P(x)-1$ нь $(x+1)^3$-д хуваагдана гэсэн нөхцөлөөс дахин 3 тэнцэтгэл гаргаж авна. Олсон 6 тэгшитгэл бүхий системийг бодоод $a=-\dfrac38$, $c=\dfrac54$, $e=-\dfrac{15}8$ гэж олно.

Бодлогын бүтэн өгүүлбэр. $18\sqrt3+35i$ тооны куб язгуураас гурван тоо гарахын нэг нь $2\sqrt3+i$ юм. Нөгөө хоёр тоог ол.
Бодолт 1.
8x100 (www.iMath.mn)-н бодолт
Нэгжийн куб язгуур болох $$w=\dfrac12(-1+\sqrt3i)$$ $$w^2=\dfrac12(-1-\sqrt3i)$$ тоонуудаар үржүүлэхэд гарах $zw,\ zw^2$ тоонууд нь нөгөө язгуурууд нь болно. Өөрөөр хэлбэл \begin{align*} zw&=\dfrac12(2\sqrt3+i)(-1+\sqrt3i)=\\ &=\dfrac12(-2\sqrt3-i+6i-\sqrt3)=\\ &=\dfrac12(-3\sqrt3+5i) \end{align*} \begin{align*} zw^2&=\dfrac12(2\sqrt3+i)(-1-\sqrt3i)=\\ &=\dfrac12(-2\sqrt3-i-6i+\sqrt3)=\\ &=\dfrac12(-\sqrt3-7i) \end{align*} тоонууд бодлогын хариу болно.
Бодолт 2.
Д. Ариунтуяа (УБ, БЗД, 14-р сургууль 12ё анги)-н бодолт
$z^3=18\sqrt3+35i$ гэе. $z_1=2\sqrt3+i$ бөгөөд $$z^3-(2\sqrt3+i)^3=0\Rightarrow$$ $$\left[\begin{array}l z-2\sqrt3-i=0\\ z^2+(2\sqrt3+i)z+(2\sqrt3+i)^2=0\end{array}\right.$$ байна. Хоёр дахь тэгшитгэлийн шийдүүд нь $$z_2=\dfrac{-3\sqrt3+5i}2$$ $$z_3=\dfrac{-\sqrt3-7i}2$$ бөгөөд эдгээр нь бидний олох ёстой тоонууд юм.

Бодлогын бүтэн өгүүлбэр. $\{a_n\}$ бүхэл тоон дарааллын $n$-р гишүүн нь $a_n\approx\sqrt n$ гэж тодорхойлогдов. Тэгвэл $$\dfrac1{a_1}+\dfrac1{a_2}+...+\dfrac1{a_{2023}}$$ илэрхийллийн утгыг ол.
Бодолт 1.
8x100 (www.iMath.mn)-н бодолт
$n^2-n+1\le k\le n^2+n$ байх $k$ тоо яг $2n$ ширхэг байх бөгөөд эдгээрийн хувьд $$\left(n-\dfrac12\right)^2< k< \left(n+\dfrac12\right)^2$$ байна. Өөрөөр хэлбэл эдгээр $k$-ийн хувьд $a_k=n$ байна гэсэн үг юм.
Иймд, ямар нэг $m$ тооны хувьд $\dfrac1m$ хэлбэрийн нэмэгдэхүүн $2m$ ширхэг байна. Тодоруулж бичвэл $$1+\underbrace{\dfrac12+\dfrac12}_{2\cdot2}+\underbrace{\dfrac13+...+\dfrac13}_{2\cdot3}+...$$ $$...+\underbrace{\dfrac1m+...+\dfrac1m}_{2m}+...$$ байна.
$44^2+44=1980< 2023$ тул $$\dfrac1{a_1}+\dfrac1{a_2}+...+\dfrac1{a_{2023}}=$$ $$=2\cdot44+\left(\dfrac1{45}+...\dfrac1{45}\right)\cdot43=$$ $$=88+\dfrac{43}{45}$$ гарна.

Бодлогын бүтэн өгүүлбэр. $f_1(x)=(x-2)^2$ ба $f_n(x)=(f_{n-1}(x)-2)^2$ гэж тодорхойлогдсон бол $f_{2023}(x)$ функцийн $x^2$-н өмнөх коэффициентийг ол.
Бодолт 1.
8x100 (www.iMath.mn)-н бодолт
Бид $x^2$-н өмнөх коэффициентийг олох шаардлагатай тул дарааллын гишүүн бүр дэх $x^2$, $x$ болон сул гишүүдийг судлах хэрэгтэй. Иймд $$f_n(x)=...+a_nx^2-b_nx+c_n$$ гэж тэмдэглэе. $a_1=1$, $b_1=4$ ба $\forall n\in\mathbb N:\ c_n=4$ байх нь ойлгомжтой юм. Иймд, $$b_{n+1}=4b_n=4^n$$ бөгөөд $$a_{n+1}=4a_n+b_n^2=4a_n+16^n$$ байхыг хялбархан шалгаж болно. Эндээс $$a_n=\dfrac{16^n-4^n}{12}$$ гэж олдох тул бодлогын хариу $$a_{2023}=\dfrac{16^{2023}-4^{2023}}{12}$$ байна.

Бодлогын бүтэн өгүүлбэр. $(3+\sqrt7)^{2023}$ тооны бүхэл хэсэг тэгш байх уу? сондгой байх уу? $a$ тооноос үл хэтрэх хамгийн их бүхэл тоог $a$ тооны бүхэл хэсэг гэж нэрлэдэг.
Бодолт 1.
8x100 (www.iMath.mn)-н бодолт
Д. Ариунтуяа (14-р сургуулийн 12ё анги)
О. Лхагва (Увс, Тариалан сум 1 сургууль, 11 а анги) нарын бодолтын санаа доорх бодолттой ижил байсан.

Ерөнхий тохиолдолд авч үзье. Дурын натурал $n$ тооны хувьд $$(3+\sqrt7)^n=A+B\sqrt7$$ $$(3-\sqrt7)^n=A-B\sqrt7$$ байх нь ойлгомжтой. Энд, $A,B\in\mathbb N$ юм. Эндээс $$(3+\sqrt7)^n+(3-\sqrt7)^n=2A\qquad(*)$$ байна. Өөрөөр хэлбэл, тэгш тоо гарна.

$(3+\sqrt7)(3-\sqrt7)=2$ тул $$(3-\sqrt7)^n=\dfrac{2^n}{(3+\sqrt7)^n}< 1$$ буюу $$0< 1-(3-\sqrt7)^n< 1$$ байна. $(*)$-ээс $$(3+\sqrt7)^n=2A-(3-\sqrt7)^2=$$ $$=2A-1+\underbrace{1-(3-\sqrt7)^2}_{\mbox{бутархай хэсэг}}$$ болох тул $$\left[(3+\sqrt7)^n\right]=2A-1$$ байна. Өөрөөр хэлбэл, $n$-ээс үл хамаараад үргэлж сондгой байна.

Бодолт 2.
Т. Базар (R3 сургалтын төв)-н бодолт
$3+\sqrt7=a$, $3-\sqrt7=b$ гэе. $x^2-6x+2=0$ тэгшитгэлийн шийдүүд $a,b$ байна. $a^n+b^n=t_n$ гэвэл $$x^{n+2}=6x^{n+1}-2x^n\Rightarrow$$ $$\Rightarrow t_{n+2}=6t_{n+1}-2t_n$$ ба $t_1=6$, $t_2=32$ болно. Эндээс $\forall n: t_n\in\mathbb Z$ бөгөөд тэгш байна.
$0< b^n< 1$ гэдгээс $$[a^n]=[t_n-b^n]=t_n-1$$ сондгой болно. Энд $t_n-1< t_n-b^n< t_n$ тул $[t_n-b^n]=t_n-1.$
Өөрөөр хэлбэл, $\forall n\in\mathbb N:[(3+\sqrt7)^n]$ сондгой.

Бодлогын бүтэн өгүүлбэр. $f(x)=x^{99}+x^{98}+...+x+1$ бол $f(x^{100})$-г $f(x)$-д хуваахад гарах үлдэгдлийг ол.
Бодолт 1.
Д. Ариунтуяа (УБ, БЗД, 14-р сургууль 12ё анги)-н бодолт
$f(x)=\dfrac{x^{100}-1}{x-1}$ буюу $f(x)(x-1)+1=x^{100}$ байна. Харин, $$f(x^{100})=\dfrac{(x^{100})^{100}-1}{x^{100}-1}$$ байх бөгөөд $x^{100}=f(x)(x-1)+1$ гэж орлуулвал $$f(x^{100})=\dfrac{(f(x)(x-1)+1)^{100}-1}{f(x)(x-1)}$$ $(f(x)(x-1)+1)^{100}$-г биномын томьёогоор задалвал $$(f(x)(x-1)+1)^{100}=$$ $$=\sum_{k=0}^{100}\binom{100}k(f(x)(x-1))^{100-k}=$$ $$=\sum_{k=0}^{98}\binom{100}k(f(x)(x-1))^{100-k}+$$ $$+\binom{100}1f(x)(x-1)+1=$$ $$=Q(x)f^2(x)(x-1)^2+100f(x)(x-1)+1$$ байна. Иймд, $$f(x^{100})=Q(x)f(x)(x-1)+100$$ болох бөгөөд үлдэгдэл нь $R=100$ гэж гарна.
Бодолт 2.
Т. Базар, Ч. Гантөмөр (R3 сургалтын төв, Математикийн Тасралтгүй Боловсрол)-н бодолт

Лемм. $f(x)\equiv g(x)\pmod {h(x)}$ бол дурын $k(x)$ олон гишүүнтийн хувьд $$k(f(x))\equiv k(g(x))\pmod{h(x)}$$ байна.

Баталгаа. $f(x)\equiv g(x)\pmod {h(x)}$ бол дурын $n\in\mathbb N$ ба $a\in\mathbb Z$ тоонуудын хувьд $$a[f(x)]^n\equiv a[g(x)]^n$$ байх тул дурын $k(x)=\sum\limits_{n=0}^N a_nx^n$ олон гишүүнтийн хувьд $$k(f(x))=\sum_{n=0}^Na_n[f(x)]^n\equiv$$ $$\equiv \sum_{n=0}^Na_n[g(x)]^n=k(g(x))\pmod{h(x)}$$ болж батлагдав.

$x^{100}=f(x)(x-1)+1$ буюу $$x^{100}\equiv 1\pmod{f(x)}$$ тул лемм ёсоор $$f(x^{100})\equiv f(1)=100\pmod{f(x)}$$ байна.

Бодолт 3.
Д. Ганзориг (Олонлог төв сургууль)-н бодолт
$x^{100}=(x-1)f(x)+1$ байна. Иймд $$f(x^{100})=f((x-1)f(x)+1)=$$ $$=\sum_{k=0}^{99}((x-1)f(x)+1)^{99-k}=$$ $$=f(x)P(x)+100$$ байх $P(x)$ олон гишүүнт олдоно. Өөрөөр хэлбэл, $f(x^{100})$-г $f(x)$-д хуваахад гарах үлдэгдэл нь $100$ юм.
Бодолт 4.
Д. Ганзориг (Олонлог төв сургууль)-н бодолт
Дурын натурал $n$ тооны хувьд $a-b|a^n-b^n$ байна. $$f(x^{100})-100=$$$$=\sum_{k=0}^{99}\left[(x^{100})^k-1^k\right]$$ гэж бичвэл нэмэгдхүүн тус бүр нь $x^{100}-1$-д хуваагдана. Мөн $f(x)|x^{100}-1$ тул $$f(x)\big|f(x^{100})-100$$ болох тул $f(x^{100})$-г $f(x)$-д хуваахад гарах үлдэгдэл нь $100$ юм.
Бодолт 5.
Д. Ганзориг (Олонлог төв сургууль)-н бодолт
$f(x)=\dfrac{x^{100}-1}{x-1}$ тул $f(x)=0$ тэгшитгэл $$\varepsilon_k=\cos\dfrac{2\pi k}{100}+i\sin\dfrac{2\pi k}{100},\ k =\overline{1,99}$$ нэгжийн язгууруудтай байх буюу $$f(x)=\prod_{k=1}^{99}(x-\varepsilon_k)$$ байна. $\forall k\in\{1,...,99\}$-н хувьд $\varepsilon_k^{100}=1$ тул $$f(\varepsilon_k^{100})-100=$$ $$=f(1)-100=0$$ байна. Өөрөөр хэлбэл $\varepsilon_k$, $k=\overline{1,99}$ тоонууд $f(x^{100})-100=0$ тэгшитгэлийн язгуур болно. Иймд $$f(x)\big|f(x^{100})-100$$ буюу $f(x^{100})$-г $f(x)$-д хуваахад гарах үлдэгдэл нь $100$ гэж гарна.

Хэрвээ, чи хичээвэл юунд ч хүрч чадна...
- 8x100
Сэлүүн Санаа ХХК-ийн дэргэдэх 8x100 сургалтын төв нь үйл ажиллагаагаа эхлэсний 5 жилийн ойгоо тохиолдуулан олон арга хэмжээ зохиож байгаагийн нэг нь энэ цахим сургалтын систем болно.

Ирээдүй, хойч үеийн эзэд болох сурагч та бүхнийг хүссэн мэргэжилээ саадгүй сонгож, итгэл үнэмшил дүүрэн сурч, илүү өндөр мэдлэгтэй, чадварлаг боловсон хүчин болж эх орондоо зүтгээсэй гэсэн хүслийн дор ийнхүү ажиллаж байна.